Đến nội dung

Hình ảnh

Sáng tạo trong bất đẳng thức

* * * * - 3 Bình chọn

  • Please log in to reply
Chủ đề này có 25 trả lời

#1
alex_hoang

alex_hoang

    Thượng úy

  • Hiệp sỹ
  • 1152 Bài viết

SÁNG TẠO TRONG BẤT ĐẲNG THỨC

Chào các bạn ,để cho box bất đẳng thức Olympic được sôi nổi hơn và để thoả mãn thú vui sáng tác của các cao thủ ở VMF mình xin phép được lập chủ đề này với hi vọng tạo được một sân chơi tốt về bất đẳng thức và cùng nhau thoả sức sáng tác:)
Các bạn hãy ghi rõ tên của các bạn sau mỗi bài post để sau này nếu có tổng hợp lại cũng có thể dễ dàng hơn :)
Bài 1:Cho các số thực dương $a,b,c$ thoả mãn $a+b+c=3$.Chứng minh rằng
\[\frac{{{a^2} + 1}}{{{b^2} + ac}} + \frac{{{b^2} + 1}}{{{c^2} + ab}} + \frac{{{c^2} + 1}}{{{a^2} + bc}} \ge 3\]

Vũ Huy Hoàng


alex_hoang


HẸN NGÀY TRỞ LẠI VMF THÂN MẾN

http://www.scribd.co...oi-Ban-Cung-The

#2
Ispectorgadget

Ispectorgadget

    Nothing

  • Quản lý Toán Phổ thông
  • 2946 Bài viết
Bài 2: Cho a,b,c thực dương. Tìm GTNN của biểu thức
$$K=(\frac{ac}{ac+2b^2})^{\frac{5}{2}}+(\frac{bc}{bc+2a^2})^{\frac{5}{2}}+(\frac{ab}{ab+2c^2})^{\frac{5}{2}}$$

Trần Trung Kiên


Bài viết đã được chỉnh sửa nội dung bởi Ispectorgadget: 04-06-2012 - 22:07

►|| The aim of life is self-development. To realize one's nature perfectly - that is what each of us is here for. ™ ♫


#3
tranghieu95

tranghieu95

    Trung sĩ

  • Thành viên
  • 147 Bài viết

Bài 2: Cho a,b,c thực dương. Tìm GTNN của biểu thức
$$K=(\frac{ac}{ac+2b^2})^{\frac{5}{2}}+(\frac{bc}{bc+2a^2})^{\frac{5}{2}}+(\frac{ab}{ab+2c^2})^{\frac{5}{2}}$$

Trần Trung Kiên

Áp dụng bđt Bernoulli ta có:
$(\dfrac{3ac}{ac+2b^2})^{\frac{5}{2}} \geq \dfrac{5}{2}.\left ( \dfrac{3ac}{ac+2b^2} \right )+1-\dfrac{5}{2}$
Tương tự ta đc:
$VT.\sqrt{3^5} \geq \dfrac{5}{2}.(\dfrac{3ac}{ac+2b^2}+\dfrac{3bc}{bc+2a^2}+\dfrac{3ab}{ab+2c^2})-\dfrac{9}{2}
\geq \dfrac{15}{2}.(\dfrac{(ab+bc+ca)^2}{a^2b^2+b^2c^2+c^2a^2+6abc}-\dfrac{9}{2}=3$
$\Rightarrow VT \geq \dfrac{1}{\sqrt{3}}$
Dấu $=$ khi và chỉ khi $a=b=c$

Bài viết đã được chỉnh sửa nội dung bởi tranghieu95: 04-06-2012 - 22:34

TỪ TỪ LÀ HẠNH PHÚC
A1K39PBC

#4
le_hoang1995

le_hoang1995

    Sĩ quan

  • Thành viên
  • 314 Bài viết
Hy vọng bài này không trùng với 1 bài toán của tác giả khác.
Bài 3 ( Lê Văn Hoàng): Cho các số a,b,c,d,e không âm. CMR:
$$\left ( \frac{a+b+c+d}{4} \right )^4+e.\left ( \frac{a+b+c+d+e}{5} \right )^3\geq 2.\left ( \frac{a+b+c+d+e}{5} \right )^4$$
Bài toán tổng quát vẫn đúng.

Bài viết đã được chỉnh sửa nội dung bởi le_hoang1995: 04-06-2012 - 23:21


#5
Nguyenhuyen_AG

Nguyenhuyen_AG

    Trung úy

  • Thành viên nổi bật 2016
  • 945 Bài viết

Hy vọng bài này không trùng với 1 bài toán của tác giả khác.
Bài 3 ( Lê Văn Hoàng): Cho các số a,b,c,d,e không âm. CMR:
$$\left ( \frac{a+b+c+d}{4} \right )^4+e.\left ( \frac{a+b+c+d+e}{5} \right )^3\geq 2.\left ( \frac{a+b+c+d+e}{5} \right )^4$$
Bài toán tổng quát vẫn đúng.


Bài này có hình thức nhìn lạ mắt nhỉ, lời giải này mình nghĩ chắc cũng là con đường mà tác giả sáng tác ra bài toán.
Ta viết bất đẳng thức cần chứng minh lại như sau $$\left ( \frac{a+b+c+d}{4} \right )^4\geq 2.\left ( \frac{a+b+c+d+e}{5} \right )^4-e.\left ( \frac{a+b+c+d+e}{5} \right )^3,$$ $$\left ( \frac{a+b+c+d}{4} \right )^4\geq \left ( \frac{a+b+c+d+e}{5} \right )^3\left [\frac{2\left (a+b+c+d \right )-3e}{5}\right ]$$ Nếu $\frac{2\left (a+b+c+d \right )-3e}{5}<0$ thì ta có $VT\ge 0\ge VP.$ Xét trường hợp ngược lại, sử dụng bất đẳng thức AM-GM, ta có $$\begin{aligned} \left ( \frac{a+b+c+d+e}{5} \right )^3\left [\frac{2\left (a+b+c+d \right )-3e}{5}\right ]&\le\left ( \frac{\dfrac{3\left (a+b+c+d+e \right )}{5}+\dfrac{2\left (a+b+c+d \right )-3e}{5}}{4}\right )^4\\&=\left ( \frac{a+b+c+d}{4} \right )^4.\end{aligned}$$ Bài toán được chứng minh xong. $\Box$

Bài 4. [Nguyenhuyen_AG] Với $a,b,c$ là ba số thực sao cho $3a^2+2b^2+c^2=6.$ Hãy tìm giá trị lớn nhất và nhỏ nhất của biểu thức $$F(a,b,c)=2(a+b+c)-abc.$$
P/s. Số đẹp 3,2,1, 3+2+1=6, F giống VMO-2002.

Bài viết đã được chỉnh sửa nội dung bởi Nguyenhuyen_AG: 05-06-2012 - 12:19

Nguyen Van Huyen
Ho Chi Minh City University Of Transport

#6
Tham Lang

Tham Lang

    Thượng úy

  • Thành viên
  • 1149 Bài viết
Bài toán 5. [ huymit_95 ]
Cho $a,b,c$ là các số thực dương. Chứng minh rằng :
$$\sqrt{\dfrac{a+b}{c}}+\sqrt{\dfrac{b+c}{a}}+\sqrt{\dfrac{c+a}{b}} \ge \sqrt[6]{\dfrac{2304(a+b+c)^4\sqrt{a+b+c}}{\sqrt{3abc}(a+b)(b+c)(c+a)}}$$

Bài viết đã được chỉnh sửa nội dung bởi huymit_95: 22-06-2012 - 16:38

Off vĩnh viễn ! Không ngày trở lại.......


#7
tim1nuathatlac

tim1nuathatlac

    Thượng sĩ

  • Thành viên
  • 298 Bài viết
cho a, b, c là các số không âm.CMR
$\sum \sqrt{\frac{a^{2}+4bc}{b^{2}+c^{2}}}\geq 2+\sqrt{2}$


#8
daothanhoai

daothanhoai

    Trung sĩ

  • Thành viên
  • 160 Bài viết
Bài 1: http://diendantoanho...i1na-ib-jright/

Bài 2: Hai bộ số $(x)=(x_1,x_2,...,x_n)$ và $(y)=(y_1,y_2,...,y_n)$ thỏa mãn 3 tính chất sau thì bộ (x) gọi là trội hơn bộ (y)

1. $x_1\geq x_2\geq ...\geq x_n$ và $y_1\geq y_2\geq ...\geq y_n$
2. $x_1+x_2+...+x_i\geq y_1+y_2+..+y_i, \vee i=1,2,...,(n-1)$
3.$x_1+x_2+...+x_n = y_1+y_2+..+y_n$
ta ký hiệu: $(x)\succ y$

Tiếp theo ta cho hai bộ số: $(a)=(a_1,a_2,...,a_n)$ trội hơn bộ số $(b)=(b_1,b_2,...,b_n)$.

Khi đó ta có bất đẳng thức sau:

$a_1x_1+a_2x_2+...a_nx_n \geq b_1y_1+b_2y_2+...b_ny_n $

Mở rộng khi có các cặp bộ trội dương như sau:

Bộ các bộ $(a_j)\succ (b_j) $ trong đó:

$(a_1)=(a_{11},a_{12},...a_{1n}); b_1=(b_{11},b_{12},...b_{1n})$
$(a_2)=(a_{21},a_{22},...a_{2n}); b_1=(b_{21},b_{22},...b_{2n})$
$...............................................$
$(a_m)=(a_{m1},a_{m2},...a_{mn}); b_1=(b_{m1},b_{m2},...b_{mn})$

Với $a_{ij}>0; b_{ij}>0$
Ta sẽ có một bất đẳng thức tổng quát như sau:

$\sum_{j=1}^{n}\left ( \prod_{i=1}^{m}a_{ji} \right )\geq \sum_{j=1}^{n}\left ( \prod_{i=1}^{m}b_{ji} \right )$

(Hi vọng không trùng với bất đẳng thức nào của tác giả khác-nếu trùng sẽ xóa- đang ngờ ngợ trùng với BĐT muihaird may không phải. Anh đã chứng minh được rùi các em học sinh thử chứng minh xem; nếu không chứng minh được anh sẽ chứng minh)

Bài 3: (Daothanhoai và Lehoang_95)

Với $a,b,c,r\geq 0$ chứng minh bất đẳng thức sau

$a^r(a^2-b^2)(c+a)+b^r(b^2-c^2)(a+b)+c^r(c^2-a^2)(b+c)\geq 0$

Chứng minh tại đây:

http://diendantoanho...crc2-a2bcgeq-0/

Bài 4: (daothanhoai và Lehoang_95)

$a_i>1; i=1,2,...,n$ đặt
$x=\frac{a_1+a_2+...+a_n}{n}$

Ta có:

$a_1^{a_1}+a_2^{a_2}+....+a_n^{a_n}\geq a_1^x+a_2^x+...+a_n^x$

Chứng minh tại đây:

http://diendantoanho...-thức-karamata/





Đào Thanh Oai


Bài viết đã được chỉnh sửa nội dung bởi daothanhoai: 01-08-2012 - 18:42


#9
daothanhoai

daothanhoai

    Trung sĩ

  • Thành viên
  • 160 Bài viết
Khi hàm f(x) lồi trên $I(a,b)$, với ${{x}_{i}}\in I(a,b),i=\overline{1,n}$

${{J}_{k}} \ge {{J}_{k+1}}$ (đã được chứng minh)

Với $ m>n\ge p>q, m+q=n+p $

${{J}_{m}}+{{J}_{q}}\ge {{J}_{n}}+{{J}_{p}} $ (rút ra từ việc quan sát một số dạng bất đẳng thức Popociviu chưa được chứng minh)

Bài viết đã được chỉnh sửa nội dung bởi daothanhoai: 09-08-2012 - 20:04


#10
tim1nuathatlac

tim1nuathatlac

    Thượng sĩ

  • Thành viên
  • 298 Bài viết
mình nghĩ mãi nhưng chưa nghĩ ra lời giải.mọi người giúp mình nha
cho a, b, c là các số dương và thỏa mãn abc=1.CMR
$\sum \frac{1}{a\sqrt{a+b}}\geq \frac{3}{\sqrt{2}}$
bài này cũng đúng vs moi a, b, c hay sao đấy.tích abc ở trong căn mẫu VP


#11
ntuan5

ntuan5

    Hạ sĩ

  • Thành viên
  • 93 Bài viết
Sự bđt (Nguyễn Vũ Anh Tuấn):
Cho:
$x_1,x_2,...,x_n \in [0;2]$
Hãy chứng tỏ:
$(a + x_1 ^ t) (a + x_2 ^ t) ... (a + x_n ^ t) \geq [a + \frac{(x_1+x_2+...+x_n)^t}{n ^ t}] ^ n$
Với: $a\geq1;t, \geq 2 $

Bài viết đã được chỉnh sửa nội dung bởi ntuan5: 08-08-2012 - 10:58


#12
daothanhoai

daothanhoai

    Trung sĩ

  • Thành viên
  • 160 Bài viết
${{a}_{i}}\ge 0,{{x}_{i}}>0,i=\overline{1,n}$
${{x}_{1}}+{{x}_{2}}+...+{{x}_{n}}=n$;
${{a}_{1}}\ge {{a}_{2}}\ge ....\ge {{a}_{n}}$;
${{x}_{1}}\ge {{x}_{2}}\ge ....\ge {{x}_{n}}$

Chứng minh rằng

$\left( {{a}_{1}}+{{x}_{1}} \right)\left( {{a}_{2}}+{{x}_{2}} \right)...\left( {{a}_{n}}+{{x}_{n}} \right)\le ({{a}_{1}}+1)({{a}_{2}}+1)...({{a}_{n}}+1)$(đã chứng minh-Đào Thanh Oai)

${{a}_{1}}^{{{x}_{1}}}+{{a}_{2}}^{{{x}_{2}}}+.....+{{a}_{n}}^{{{x}_{n}}}\ge {{a}_{1}}+{{a}_{2}}+...+{{a}_{n}}$ (đã chứng minh-Đào Thanh Oai-Lê Văn Hoàng)

Bài viết đã được chỉnh sửa nội dung bởi daothanhoai: 04-08-2012 - 16:04


#13
phudinhgioihan

phudinhgioihan

    PĐGH$\Leftrightarrow$TDST

  • Biên tập viên
  • 348 Bài viết

Bài thứ $n$ (Trương Tấn Đạt) (đơn giản mà hiệu quả)

Cho $n$ số thực không âm $a_1 \ge a_2 \ge...\ge a_n $ thỏa $a_1+a_2+...+a_n=n$.

Cho $\{x_n\}_1^{+\infty} \subset [0;1] $ là dãy tăng.



Chứng minh: $$\sum_{i=1}^na_i^{x_i} \le n $$


Bài viết đã được chỉnh sửa nội dung bởi phudinhgioihan: 01-01-2013 - 16:46

Phủ định của giới hạn Hình đã gửi

Đó duy sáng tạo ! Hình đã gửi


https://phudinhgioihan.wordpress.com/

#14
nguyensidang

nguyensidang

    Binh nhất

  • Thành viên
  • 32 Bài viết
E mới vào diễn đàn, e xin đưa bài này:
Cho 3 số thực dương a,b,c.Thoả mãn $a^3+b^3+c^3=3$. Chứng minh rằng:
$$ab(c-2)(c+1)+bc(a+2)(a-1)+ca(b+2)(b-2) \leq 0$$

Bài viết đã được chỉnh sửa nội dung bởi dark templar: 09-03-2013 - 17:24


#15
mai dsung

mai dsung

    Binh nhất

  • Thành viên
  • 38 Bài viết

Cho a, b, c là các số thực không âm thỏa mãn $a^{2}+b^{2}+c^{2}=1$. Tìm giá trị lớn nhât của biểu thức:

P=(a-b)(b-c)(c-a)(a+b+c)



#16
25 minutes

25 minutes

    Thành viên nổi bật 2015

  • Hiệp sỹ
  • 2795 Bài viết

Cho a, b, c là các số thực không âm thỏa mãn $a^{2}+b^{2}+c^{2}=1$. Tìm giá trị lớn nhât của biểu thức:

P=(a-b)(b-c)(c-a)(a+b+c)

Để biểu thức $P \geq 0$ ta có các trường hợp sau:

+) $a \geq c \geq b$.

+) $b \geq a \geq c$.

+) $c \geq b \geq a$.

Như vậy ta chỉ cần xét một trường hợp thì cũng đúng cho hai trường hợp còn lại.

Ta xét: $a \geq c \geq b \Rightarrow a^2=1-b^2-c^2 \geq 1-2a^2 \Rightarrow a \geq \dfrac{1}{ \sqrt{3}}$

Ta có: $ a^2+b^2+c^2=1 \Rightarrow b^2=1-a^2-c^2 \le 1-2b^2 \Rightarrow b \le \dfrac{1}{ \sqrt{3}}$

Và ta lại có: $a^2+c^2=1-b^2 \le 1 \Rightarrow a^2 \le 1-c^2 \Rightarrow a \le \sqrt{1-c^2}$

Và lưu ý: $a \le 1$

Đối với trường hợp này ($a \geq c \geq b$) 90% dự đoán điểm rơi sẽ là $b=0$ nên ta sẽ đi theo hướng khảo sát hàm số $P(b)$ ( Quan trọng là miền của $b$ ta phải xác định càng kĩ càng tốt vì đây là yếu tố quyết định, chỉ cần miền của $b$ lớn hơn một tí là chúng ta sẽ thấy hướng đi không đảm bảo khi khảo sát. Ở đây có hai miền của $b$ mà chúng ta cần nghĩ đến là $0 \le b \le \dfrac{1}{ \sqrt{3}}$ và $0 \le b \le c$. Tuy nhiên với cách làm của tôi thì các bạn sẽ thấy ta nên chọn miền nào của $b$.

Sau đây là lời giải:

Ta có: $P=(a-c) \big[ 2b^3-(1+ac)b+a^2c+ac^2 \big]$

Ta xét hàm số $P(b)=(a-c) \big[ 2b^3-(1+ac)b+a^2c+ac^2 \big]$

Ta có: $P^{'}(b) =(a-c)(6b^2-1-ac)=0 \Rightarrow b=- \sqrt{ \dfrac{1+ac}{6}}, b= \sqrt{ \dfrac{1+ac}{6}}$

Rõ ràng nếu ta dùng miền $b$ là $0 \le b \le c$ thì $b= \sqrt{ \dfrac{1+ac}{6}}$ chưa thể $ \le c$. Nhưng khi dùng miền $0 \le b \le \dfrac{1}{ \sqrt{3}}$ thì $b= \sqrt{ \dfrac{1+ac}{6}} \le \dfrac{1}{ \sqrt{3}}$ luôn đúng và $P(0)>P \Bigg( \dfrac{1}{ \sqrt{3}} \Bigg)$. Vậy nên ta chọn miền $0 \le b \le \dfrac{1}{ \sqrt{3}}$.

Lập bảng biến thiên ta có $MaxP= P(0)=ac(a-c)(a+c)=ac(a^2-c^2)$

Ta lại có: $P(0) \le c \sqrt{1-c^2}(1-2c^2)$

Đặt $F(c)=c \sqrt{1-c^2}(1-2c^2)$ với $c \in \big[0; \dfrac{ \sqrt{2}}{2} \big]$.

Giải thích: $c \in \big[0; \dfrac{ \sqrt{2}}{2} \big]$ vì $c^2 \le a^2=1-b^2-c^2 \le 1-c^2 \Rightarrow c^2 \le 1-c^2 \Rightarrow 0 \le c \le \dfrac{ \sqrt{2}}{2}$.

Ta có: $F^{'}(c)= \dfrac{8c^4-8c^2+1}{ \sqrt{1-c^2}} =0 \Rightarrow c=- \dfrac{ \sqrt{2+ \sqrt{2}}}{2}, c=- \dfrac{ \sqrt{2- \sqrt{2}}}{2}, c= \dfrac{ \sqrt{2- \sqrt{2}}}{2}, c= \dfrac{ \sqrt{2+ \sqrt{2}}}{2} $.

Lập bảng biến thiên ta có: $MaxF(c)= F \Bigg( \dfrac{ \sqrt{2- \sqrt{2}}}{2} \Bigg) = \dfrac{1}{4}$

Vậy ta có $MaxP= \dfrac{1}{4}$ tại $a= \dfrac{ \sqrt{2+ \sqrt{2}}}{2}, b=0, c= \dfrac{ \sqrt{2- \sqrt{2}}}{2} $.

________________________________________________________

Xét $$P^2=(a-b)^2(b-c)^2(c-a)^2(a+b+c)^2$$
Nếu đặt $p=a+b+c\ge 0,\ 0\le q=ab+bc+ca\le 1,\ r=abc\ge 0$. Khi đó, giả thiết suy ra $p^2=2q+1$ và ta có
\[\begin{aligned}P^2&=(p^2q^2+8pqr-27r^2-4q^3-4p^3r)p^2\\
&=-27(2q+1)r^2-4(2q+1)\sqrt{2q+1}r+(2q+1)q^2(1-2q)=f(r)\end{aligned}\]
Ta có $f'(r)=0\iff r= \dfrac{-2\sqrt{2q+1}}{27}<0.$
Kết hợp BĐT $Schur$ suy ra $r\ge \max \left \{ 0, \dfrac{(2q-1)\sqrt{2q+1}}{4} \right \}$.
Vì hàm $f(r)$ là hàm bậc hai theo ẩn $r$ có hệ số $a<0$ và hoành độ đỉnh âm nên $f(r)$ nghịch biến trên $[0;+\infty).$
+ TH1: Nếu $\dfrac{1}{2}\le q\le 1$ thì
\[f(r)\le f \left( \dfrac{\sqrt{2q+1}(2q-1)}{4} \right ) =-218q^3+93q^2+54q-23\le 0.\]
+ TH2: Nếu $0\le q\le \dfrac{1}{2}$ thì
\[f(r)\le f \left( 0 \right ) =(1+2q)q^2(1-2q)\le \dfrac{1}{16}.\]
Do đó, \[- \dfrac{1}{4}\le P\le \dfrac{1}{4}\]
Vậy, \[\boxed{\min P= - \frac{1}{4}\Leftrightarrow a= \frac{\sqrt{2+\sqrt{2}}}{2},b= \frac{\sqrt{2-\sqrt{2}}}{2},c=0}\]
\[\color{blue}{\boxed{\max P= \frac{1}{4}\Leftrightarrow b= \frac{\sqrt{2+\sqrt{2}}}{2},c= \frac{\sqrt{2-\sqrt{2}}}{2},a=0}}\]

P/S: Trích theo lời giải của nthoangcute 


Hãy theo đuổi đam mê, thành công sẽ theo đuổi bạn.



Thảo luận BĐT ôn thi Đại học tại đây


#17
ongngua97

ongngua97

    Sĩ quan

  • Thành viên
  • 311 Bài viết

cho $a,b,c> 0$ thoả $ab+bc+ac=1$

Chứng minh rằng 

$P=\sum \frac{a}{b^{2}+c^{2}+2}\geq \frac{3\sqrt{3}}{8}$


ONG NGỰA 97. :wub: 


#18
napoleong

napoleong

    Binh nhất

  • Thành viên
  • 31 Bài viết

 

SÁNG TẠO TRONG BẤT ĐẲNG THỨC

Chào các bạn ,để cho box bất đẳng thức Olympic được sôi nổi hơn và để thoả mãn thú vui sáng tác của các cao thủ ở VMF mình xin phép được lập chủ đề này với hi vọng tạo được một sân chơi tốt về bất đẳng thức và cùng nhau thoả sức sáng tác:)
Các bạn hãy ghi rõ tên của các bạn sau mỗi bài post để sau này nếu có tổng hợp lại cũng có thể dễ dàng hơn :)
Bài 1:Cho các số thực dương $a,b,c$ thoả mãn $a+b+c=3$.Chứng minh rằng
\[\frac{{{a^2} + 1}}{{{b^2} + ac}} + \frac{{{b^2} + 1}}{{{c^2} + ab}} + \frac{{{c^2} + 1}}{{{a^2} + bc}} \ge 3\]

Vũ Huy Hoàng

 

 

 

SÁNG TẠO TRONG BẤT ĐẲNG THỨC

Chào các bạn ,để cho box bất đẳng thức Olympic được sôi nổi hơn và để thoả mãn thú vui sáng tác của các cao thủ ở VMF mình xin phép được lập chủ đề này với hi vọng tạo được một sân chơi tốt về bất đẳng thức và cùng nhau thoả sức sáng tác:)
Các bạn hãy ghi rõ tên của các bạn sau mỗi bài post để sau này nếu có tổng hợp lại cũng có thể dễ dàng hơn :)
Bài 1:Cho các số thực dương $a,b,c$ thoả mãn $a+b+c=3$.Chứng minh rằng
\[\frac{{{a^2} + 1}}{{{b^2} + ac}} + \frac{{{b^2} + 1}}{{{c^2} + ab}} + \frac{{{c^2} + 1}}{{{a^2} + bc}} \ge 3\]

Vũ Huy Hoàng

 



#19
trantuananh9a

trantuananh9a

    Binh nhất

  • Thành viên
  • 41 Bài viết

Cho $X^2+Y^2+Z^2=\frac{1}{2}$ thì

 $\frac{1}{x^2+y^2}+\frac{1}{y^2+z^2}+\frac{1}{z^2+x^2}\geq 9$


Cực Ngu Hình


#20
Hoang Tung 126

Hoang Tung 126

    Thiếu tá

  • Thành viên
  • 2061 Bài viết

Áp dụng bdt Bunhia có:$\frac{1}{x^2+y^2}+\frac{1}{y^2+z^2}+\frac{1}{z^2+x^2}\geq \frac{9}{2(x^2+y^2+z^2)}=9$






0 người đang xem chủ đề

0 thành viên, 0 khách, 0 thành viên ẩn danh